MidtermExam (Solutions - Final)
MidtermExam (Solutions - Final)
College of Engineering
Department of Electrical & Electronics Engineering
EENG 381: Control Systems
Semester 1: 2022/2023
Midterm Exam
3rd November 2022: (12:00 - 13:15) - Hall 15
Examiners responsible: Dr. Ayman Al-Khazraji & Dr. Ghassan Murad
Student Name
I.D. No.
Section:
Question 1 10
Question 2 10
Question 3 10
Question 4 10
Instructions:
● Ensure your exam paper contains four different questions.
● Mobiles and/or tablets are not allowed.
● All answers should be clearly written on the attached sheets only.
● All answers should be your own. Students caught cheating will receive an F grade.
Some Useful Laplace Transform Pairs
f (t) F (s)
1
1 unit step 1(t) s
n!
2 tn (n = 1, 2, 3, . . . ) sn+1
1
3 e−at s+a
The configuration of a torque T (t) applied to a body having a moment of inertia J , a twisting
shaft having a stiffness factor K, and a fluid damper having a damping factor B, is shown in Figure
1. The system also produces a displacement θ(t).
θ(s)
Find the transfer function expression . [3 marks]
T (s)
K
θ(t) T (t)
J
Applying Newton’s second law of motion to J - that is, net external torque = moment of inertia
× angular acceleration - results in the following differential equation:
d2 θ(t) dθ(t)
J = T (t) − B − Kθ(t)
dt2 dt
Re-arranging we get:
d2 θ(t) dθ(t)
J +B + Kθ(t) = T (t)
dt2 dt
Taking the Laplace transform gives:
Ô⇒ (J s2 + Bs + K) θ(s) = T (s)
θ(s) 1
=
T (s) J s2 + Bs + K
Midterm - EENG 381 2
Y (s)
Simplify the block diagram shown in Figure 2 to obtain the closed-loop transfer function
R(s)
(show your steps). [7 marks]
G4
R(s) + Y (s)
G1 + + G2 + G3
- -
H1
H2
Solution:
G4
G2
R(s) + Y (s)
G1 + + G2 + G3
- -
H1
H2
R(s) G2 G4 Y (s)
G1 + 1+ G3
- 1 + G2 H1 G2
H2
1
Midterm - EENG 381 3
A ship with forward velocity u(t) and angular velocity ω(t) is represented by the following
differential equation:
du(t)
m + Bu(t) = F ω(t)
dt
Given that: m = 18 × 103 kg, B = 150 N s/m, and F = 96 N s/rad,
(b) Find an expression for the time response of the output when there is a step change of 12.5 rad/sec
in the input. [3 marks]
Solution (a):
U (s) F F
B
96
150
0.64
Ô⇒ = = = =
ω(s) ms + B m
B
s+1 18×103
s+1 120s + 1
150
where:
Solution (b):
0.64 8
120
12.5 120
U (s) = × =
s + 120
1
s s (s + 120
1
)
Using partial fractional expansion to find the inverse Laplace transform, U (s) may then be ex-
pressed as:
8
120
c1 c2
= +
s (s + 120 ) s s + 120
1 1
8 8
120 1 120
Ô⇒ c1 = lim s = 8, c2 = lim1 (s + ) = −8
s→0 s (s + 120
1
) s→− 120 120 s (s + 120
1
)
Therefore, the inverse Laplace transform becomes:
⎡8 8 ⎤⎥
⎢
u(t) = L−1 ⎢⎢ − 1 ⎥
⎥ = 8 (1 − e−0.0083t )
⎢ s s + 120 ⎥
⎣ ⎦
Midterm - EENG 381 4
R(s) K1 1 Y (s)
+
- s+1 s
1 + K2 s
Y (s)
(a) Find the transfer function of the system as shown in Figure 3, and select the gains K1 and
R(s)
K2 so that the system is critically damped with two equal poles at s = −10. [3 marks]
(b) From (a), calculate the peak overshoot percentage and settling time. [2 marks]
Solution (a):
K1
Y (s) s(s + 1) K1
= =
R(s) K1 (1 + K2 s) s2 + (1 + K1 K2 )s + K1
1+
s(s + 1)
The desired closed-loop characteristic equation is given by (s + 10)2 = s2 + 20s + 100. Therefore
by inspection, K1 = 100 and 1 + K1 K2 = 20, which implies that K2 = 0.19.
Solution (b):
. √
Furthermore, since wn = 100:
4 4
ts = = = 0.4 seconds
ζωn 10
.
Midterm - EENG 381 5
Question 3:
R(s) 2s + 1 Y (s)
+
-
s2 + 3s
(b) What are the position, velocity, and acceleration error constants? [5 marks]
(c) Determine the steady-state error resulting from a unit step (position) input. [3 marks]
Solution (a):
⎛ 2s + 1 2s + 1 ⎞
The open-loop transfer function = contains at least one pure integration; there-
⎝ s2 + 3s s(s + 3)⎠
fore, the system is of Type 1.
Solution (b):
2s + 1
KP (position constant error) = lim =∞
s→0 s(s + 3)
2s + 1 1
Kv (velocity constant error) = lim s =
s(s + 3) 3
s→0
2s + 1
Ka (acceleration constant error) = lim s2 =0
s→0 s(s + 3)
Solution (c):
The steady-state error ess resulting from a unit step input is:
1 1
ess = = =0
1 + KP 1+∞
Midterm - EENG 381 6
Question 4:
s5 + s4 + 2s3 + s2 + s + K
Use the Routh-Hurwitz stability criterion to find the range of K that determines the stability of the
system. [10 marks]
Solution:
s5 1 2 1
s4 1 1 K
s3 1 1−K 0
s2 K K 0
s1 −K 0
s0 K
Setting:
K > 0, results in two roots that exist in the right-half side of the s-plane; and
K < 0, results in three roots that exist in the right-half side of the s-plane.
f (t) F (s)